\chapter{The Frobenius element} Throughout this chapter $K/\QQ$ is a Galois extension with Galois group $G$, $p$ is an \emph{unramified} rational prime in $K$, and $\kp$ is a prime above it. Picture: \begin{center} \begin{tikzcd} K \ar[d, dash] & \supset & \OO_K \ar[d, dash] & \kp \ar[d, dash] & \OO_K/\kp \cong \FF_{p^f} \ar[d, dash] \\ \QQ & \supset & \ZZ & (p) & \FF_p \end{tikzcd} \end{center} If $p$ is unramified, then one can show there is a unique $\sigma \in \Gal(L/K)$ such that $\sigma(\alpha) \equiv \alpha^p \pmod{\kp}$ for every prime $p$. \section{Frobenius elements} \prototype{$\Frob_\kp$ in $\ZZ[i]$ depends on $p \pmod 4$.} Here is the theorem statement again: \begin{theorem}[The Frobenius element] Assume $K/\QQ$ is Galois with Galois group $G$. Let $p$ be a rational prime unramified in $K$, and $\kp$ a prime above it. There is a \emph{unique} element $\Frob_\kp \in G$ with the property that \[ \Frob_\kp(\alpha) \equiv \alpha^{p} \pmod{\kp}. \] It is called the \vocab{Frobenius element} at $\kp$, and has order $f$. \end{theorem} The \emph{uniqueness} part is pretty important: it allows us to show that a given $\sigma \in \Gal(L/K)$ is the Frobenius element by just observing that it satisfies the above functional equation. Let's see an example of this: \begin{example}[Frobenius elements of the Gaussian integers] Let's actually compute some Frobenius elements for $K = \QQ(i)$, which has $\OO_K = \ZZ[i]$. This is a Galois extension, with $G = \Zm2$, corresponding to the identity and complex conjugation. If $p$ is an odd prime with $\kp$ above it, then $\Frob_\kp$ is the unique element such that \[ (a+bi)^p \equiv \Frob_\kp(a+bi) \pmod{\kp} \] in $\ZZ[i]$. In particular, \[ \Frob_\kp(i) = i^p = \begin{cases} i & p \equiv 1 \pmod 4 \\ -i & p \equiv 3 \pmod 4. \end{cases} \] From this we see that $\Frob_\kp$ is the identity when $p \equiv 1 \pmod 4$ and $\Frob_\kp$ is complex conjugation when $p \equiv 3 \pmod 4$. \end{example} Note that we really only needed to compute $\Frob_\kp$ on $i$. If this seems too good to be true, a philosophical reason is ``freshman's dream'' where $(x+y)^p \equiv x^p + y^p \pmod{p}$ (and hence mod $\kp$). So if $\sigma$ satisfies the functional equation on generators, it satisfies the functional equation everywhere. We also have an important lemma: \begin{lemma} [Order of the Frobenius element] Let $\Frob_\kp$ be a Frobenius element from an extension $K/\QQ$. Then the order of $\Frob_\kp$ is equal to the inertial degree $f_\kp$. In particular, $(p)$ splits completely in $\OO_K$ if and only if $\Frob_\kp = \id$. \end{lemma} \begin{exercise} Prove this lemma as by using the fact that $\OO_K / \kp$ is the finite field of order $f_\kp$, and the Frobenius element is just $x \mapsto x^p$ on this field. \end{exercise} Let us now prove the main theorem. This will only make sense in the context of decomposition groups, so readers which skipped that part should omit this proof. \begin{proof} [Proof of existence of Frobenius element] The entire theorem is just a rephrasing of the fact that the map $\theta$ defined in the last section is an isomorphism when $p$ is unramified. Picture: \begin{center} \begin{asy} size(12cm); filldraw( (-4,-2)--(-4,2)--(1.5,2)--(1.5,-2)--cycle, lightblue+opacity(0.2), black); label("$G = \operatorname{Gal}(K/\mathbb Q)$", (-1,2), dir(90)); dot( (-1.1,-1.2) ); dot( (-1.4,0.9) ); dot( (-2,1.4) ); dot( (-2.7,-0.4) ); dot( (-3.1,0.2) ); dot( (-3.4,-1.6) ); filldraw(scale(0.8,1.8)*unitcircle, lightcyan+opacity(0.4), black); label("$D_{\mathfrak p}$", (0.8,2), dir(-90)); for (real y=-1.5; y<2; ++y) { dot( (0,y) ); } label("$\operatorname{Frob}_{\mathfrak p}$", (0,-1.5), dir(90)); filldraw(shift(5,0)*scale(0.8,1.8)*unitcircle, lightcyan+opacity(0.4), black); for (real y=0.5; y<2; ++y) { dot( (5,y) ); } dot("$T$", (5,-1.5), dir(45)); dot("$T^2$", (5,-0.5), dir(45)); draw( (1,0)--(4,0), Arrows ); label("$\left$", (5,1.8), dir(90)); draw( (0.2,-1.5)--(4.8,-1.5), dashed, EndArrow); label("$\theta(\operatorname{Frob}_{\mathfrak p}) = T$", (2.8,-1.5), dir(-90)); label("$\theta$", (2.5,0), dir(90)); label("$\cong$", (2.5,0), dir(-90)); \end{asy} \end{center} In here we can restrict our attention to $D_\kp$ since we need to have $\sigma(\alpha) \equiv 0 \pmod \kp$ when $\alpha \equiv 0 \pmod \kp$. Thus we have the isomorphism \[ D_\kp \taking\theta \Gal\left( (\OO_K/\kp) / \FF_p \right). \] But we already know $\Gal\left( (\OO_K/\kp)/\FF_p \right)$, according to the string of isomorphisms \[ \Gal\left( (\OO_K/\kp) / \FF_p \right) \cong \Gal\left( \FF_{p^f} / \FF_p \right) \cong \left< T = x \mapsto x^p \right> \cong \Zc{f} . \] So the unique such element is the pre-image of $T$ under $\theta$. \end{proof} \section{Conjugacy classes} Now suppose $\kp_1$ and $\kp_2$ are \emph{two} primes above an unramified rational prime $p$. Then we can define $\Frob_{\kp_1}$ and $\Frob_{\kp_2}$. Since the Galois group acts transitively, we can select $\sigma \in \Gal(K/\QQ)$ be such that \[ \sigma(\kp_1) = \kp_2. \] We claim that \[ \Frob_{\kp_2} = \sigma \circ \Frob_{\kp_1} \circ \sigma\inv. \] Note that this is an equation in $G$. \begin{ques} Prove this. \end{ques} More generally, for a given unramified rational prime $p$, we obtain: \begin{theorem} [Conjugacy classes in Galois groups] The set \[ \left\{ \Frob_\kp \mid \kp \text{ above } p \right\} \] is one of the conjugacy classes of $G$. \end{theorem} \begin{proof} We've used the fact that $G = \Gal(K/\QQ)$ is transitive to show that $\Frob_{\kp_1}$ and $\Frob_{\kp_2}$ are conjugate if they both lie above $p$; hence it's \emph{contained} in some conjugacy class. So it remains to check that for any $\kp$, $\sigma$, we have $\sigma \circ \Frob_\kp \circ \sigma\inv = \Frob_{\kp'}$ for some $\kp'$. For this, just take $\kp' = \sigma\kp$. Hence the set is indeed a conjugacy class. \end{proof} %We denote the conjugacy class by the \vocab{Frobenius symbol} %\[ \left( \frac{K/\QQ}{p} \right). \] In summary, \begin{moral} $\Frob_{\kp}$ is determined up to conjugation by the prime $p$ from which $\kp$ arises. \end{moral} So even though the Gothic letters look scary, the content of $\Frob_{\kp}$ really just comes from the more friendly-looking rational prime $p$. \begin{example} [Frobenius elements in $\QQ(\cbrt2,\omega)$] With those remarks, here is a more involved example of a Frobenius map. Let $K = \QQ(\cbrt2, \omega)$ be the splitting field of \[ t^3-2 = (t-\cbrt2)(t-\omega\cbrt2)(t-\omega^2\cbrt2). \] Thus $K/\QQ$ is Galois. We've seen in an earlier example that \[ \OO_K = \ZZ[\eps] \quad\text{where}\quad \eps \text { is a root of } t^6+3t^5-5t^3+3t+1. \] Let's consider the prime $5$ which factors (trust me here) as \[ (5) = (5, \eps^2+\eps+2)(5, \eps^2+3\eps+3)(5, \eps^2+4\eps+1) = \kp_1 \kp_2 \kp_3. \] Note that all the prime ideals have inertial degree $2$. Thus $\Frob_{\kp_i}$ will have order $2$ for each $i$. Note that \[ \Gal(K/\QQ) = \text{permutations of } \{\cbrt2,\omega\cbrt2,\omega^2\cbrt2\} \cong S_3. \] In this $S_3$ there are $3$ elements of order two: fixing one root and swapping the other two. These correspond to each of $\Frob_{\kp_1}$, $\Frob{\kp_2}$, $\Frob_{\kp_3}$. In conclusion, the conjugacy class $\left\{ \Frob_{\kp_1}, \Frob_{\kp_2}, \Frob_{\kp_3} \right\}$ associated to $(5)$ is the cycle type $(\bullet)(\bullet \; \bullet)$ in $S_3$. \end{example} \section{Chebotarev density theorem} Natural question: can we represent every conjugacy class in this way? In other words, is every element of $G$ equal to $\Frob_\kp$ for some $\kp$? Miraculously, not only is the answer ``yes'', but in fact it does so in the nicest way possible: the $\Frob_\kp$'s are ``equally distributed'' when we pick a random $\kp$. \begin{theorem} [Chebotarev density theorem over $\QQ$] Let $C$ be a conjugacy class of $G = \Gal(K/\QQ)$. The density of (unramified) primes $p$ such that $\{ \Frob_\kp \mid \kp \text{ above } p \} = C$ %\[ \left( \frac{K/\QQ}{p} \right) = C \] is exactly $\left\lvert C \right\rvert / \left\lvert G \right\rvert$. In particular, for any $\sigma \in G$ there are infinitely many rational primes $p$ with $\kp$ above $p$ so that $\Frob_{\kp} = \sigma$. \end{theorem} By density, I mean that the proportion of primes $p \le x$ that work approaches $\frac{\left\lvert C \right\rvert}{\left\lvert G \right\rvert}$ as $x \to \infty$. Note that I'm throwing out the primes that ramify in $K$. This is no issue, since the only primes that ramify are those dividing $\Delta_K$, of which there are only finitely many. In other words, if I pick a random prime $p$ and look at the resulting conjugacy class, it's a lot like throwing a dart at $G$: the probability of hitting any conjugacy class depends just on the size of the class. \begin{center} \begin{asy} size(8cm); bigbox("$G$"); pen b = lightcyan + opacity(0.4); pen k = black; filldraw( (-2.6,2.5)--(0.6,2.5)--(0.6,0.5)--(-2.6,0.5)--cycle, b, k); filldraw( (-2.6,-2.5)--(0.6,-2.5)--(0.6,-0.5)--(-2.6,-0.5)--cycle, b, k); filldraw( (2,0)--(3.5,0)--(3.5,2.5)--(2,2.5)--cycle, b, k); filldraw( (2,-1)--(3.5,-1)--(3.5,-2)--(2,-2)--cycle, b, k); for (real x = -2; x < 1; ++x) { dot( (x, 1.9) ); dot( (x, 1.1) ); dot( (x, -1.9) ); dot( (x, -1.1) ); } label("$37.5\%$", (-2.6, 0.5), dir(140)); label("$37.5\%$", (-2.6,-2.5), dir(140)); label("$C_1$", (-2.6, 2.5), dir(225)); label("$C_2$", (-2.6, -.5), dir(225)); dot( (2.75, 2.0) ); dot( (2.75, 1.25) ); dot( (2.75, 0.50) ); dot( (2.75, -1.50) ); label("$C_3$", (2, 0), dir(-90)); label("$18.75\%$", (3, 0), dir(-75)); label("$C_4$", (2, -2), dir(-90)); label("$6.25\%$", (3, -2), dir(-75)); \end{asy} \end{center} \begin{remark} Happily, this theorem (and preceding discussion) also works if we replace $K/\QQ$ with any Galois extension $K/F$; in that case we replace ``$\kp$ over $p$'' with ``$\kP$ over $\kp$''. In that case, we use $\Norm(\kp) \le x$ rather than $p \le x$ as the way to define density. \end{remark} \section{Example: Frobenius elements of cyclotomic fields} Let $q$ be a prime, and consider $L = \QQ(\zeta_q)$, with $\zeta_q$ a primitive $q$th root of unity. You should recall from various starred problems that \begin{itemize} \ii $\Delta_L = \pm q^{q-2}$, \ii $\OO_L = \ZZ[\zeta_q]$, and \ii The map \[ \sigma_n : L \to L \quad\text{by}\quad \zeta_q \mapsto \zeta_q^n \] is an automorphism of $L$ whenever $\gcd(n,q)=1$, and depends only on $n \pmod q$. In other words, the automorphisms of $L/\QQ$ just shuffle around the $q$th roots of unity. In fact the Galois group consists exactly of the elements $\{\sigma_n\}$, namely \[ \Gal(L/\QQ) = \{ \sigma_n \mid n \not\equiv 0 \pmod q \}. \] As a group, \[ \Gal(L/\QQ) = \Zm q \cong \Zcc{q-1}. \] \end{itemize} This is surprisingly nice, because \textbf{elements of $\Gal(L/\QQ)$ look a lot like Frobenius elements already}. Specifically: \begin{lemma}[Cyclotomic Frobenius elements] \label{lem:cyclo_frob} In the cyclotomic setting $L = \QQ(\zeta_q)$, let $p$ be a rational unramified prime and $\kp$ above it. Then \[ \Frob_\kp = \sigma_p. \] \end{lemma} \begin{proof} Observe that $\sigma_p$ satisfies the functional equation (check on generators). Done by uniqueness. % We know $\Frob_\kp(\alpha) \equiv \alpha^p \pmod{\kp}$ by definition, % but also that $\Frob_\kp = \sigma_n$ for some $n$ % We want $n=p$; since $\sigma_n(\zeta_q)^n = \zeta_q^n$ by definition % it would be very weird if this wasn't true! % % Given $\zeta_q^n \equiv \zeta_q^p \pmod{\kp}$, it suffices to % prove that the $q$th roots of unity are distinct mod $\kp$. % Look at the polynomial $F(x) = x^q-1$ in $\ZZ[\zeta_p]/\kp \cong \FF_{p^f}$. % Its derivative is \[ F'(x) = qx^{q-1} \not\equiv 0 \pmod{\kp} \] % (since $\FF_{p^f}$ has characteristic $p \nmid q$). % The only root of $F'$ is zero, hence $F$ has no double roots mod $\kp$. \end{proof} \begin{ques} Conclude that a rational prime $p$ splits completely in $\OO_L$ if and only if $p \equiv 1 \pmod q$. \end{ques} \section{Frobenius elements behave well with restriction} Let $L/\QQ$ and $K/\QQ$ be Galois extensions, and consider the setup \begin{center} \begin{tikzcd} L \ar[d, dash] & \supset & \kP \ar[d, dash] \ar[r, dotted] & \Frob_{\kP} \in \Gal(L/\QQ)\\ K \ar[d, dash] & \supset & \kp \ar[d, dash] \ar[r, dotted] & \Frob_\kp \in \Gal(K/\QQ) \\ \QQ & \supset & (p) & \end{tikzcd} \end{center} Here $\kp$ is above $(p)$ and $\kP$ is above $\kp$. We may define \[ \Frob_\kp \colon K \to K \quad\text{and}\quad \Frob_{\kP} \colon L \to L \] and want to know how these are related. \begin{theorem} [Restrictions of Frobenius elements] Assume $L/\QQ$ and $K/\QQ$ are both Galois. Let $\kP$ and $\kp$ be unramified as above. Then $\Frob_{\kP} \restrict{K} = \Frob_{\kp}$, i.e.\ for every $\alpha \in K$, \[ \Frob_\kp(\alpha) = \Frob_{\kP}(\alpha). \] \end{theorem} %\begin{proof} % We know % \[ \Frob_{\kP}(\alpha) \equiv \alpha^p \pmod{\kP} % \quad \forall \alpha \in \OO_L \] % from the definition. % \begin{ques} % Deduce that % \[ \Frob_{\kP}(\alpha) \equiv \alpha^p \pmod{\kp} % \quad \forall \alpha \in \OO_K. \] % (This is weaker than the previous statement in two ways!) % \end{ques} % Thus $\Frob_{\kP}$ restricted to $\OO_K$ satisfies the % characterizing property of $\Frob_\kp$. %\end{proof} \begin{proof} TODO: Broken proof. Needs repair. \end{proof} In short, the point of this section is that \begin{moral} Frobenius elements upstairs restrict to Frobenius elements downstairs. \end{moral} \section{Application: Quadratic reciprocity} We now aim to prove: \begin{theorem} [Quadratic reciprocity] Let $p$ and $q$ be distinct odd primes. Then \[ \left( \frac pq \right)\left( \frac qp \right) = (-1)^{\frac{p-1}{2} \cdot \frac{q-1}{2}}. \] \end{theorem} (See, e.g. \cite{ref:holden} for an exposition on quadratic reciprocity, if you're not familiar with it.) \subsection{Step 1: Setup} For this proof, we first define \[ L = \QQ(\zeta_q) \] where $\zeta_q$ is a primitive $q$th root of unity. Then $L/\QQ$ is Galois, with Galois group $G$. \begin{ques} Show that $G$ has a unique subgroup $H$ of index two. \end{ques} In fact, we can describe it exactly: viewing $G \cong \Zm q$, we have \[ H = \left\{ \sigma_n \mid \text{$n$ quadratic residue mod $q$} \right\}. \] By the fundamental theorem of Galois Theory, there ought to be a degree $2$ extension of $\QQ$ inside $\QQ(\zeta_q)$ (that is, a quadratic field). Call it $\QQ(\sqrt{q^\ast})$, for $q^\ast$ squarefree: \begin{center} \begin{tikzcd} L = \QQ(\zeta_q) \ar[d, "\frac{q-1}{2}"', dash] \ar[r, leftrightarrow] & \{1\} \ar[d, dash] \\ K = \QQ(\sqrt{q^\ast}) \ar[d, "2"', dash] \ar[r, leftrightarrow] & H \ar[d, dash] \\ \QQ \ar[r, leftrightarrow] & G \end{tikzcd} \end{center} \begin{exercise} Note that if a rational prime $\ell$ ramifies in $K$, then it ramifies in $L$. Use this to show that \[ q^\ast = \pm q \text{ and } q^\ast \equiv 1 \pmod 4. \] Together these determine the value of $q^\ast$. \end{exercise} (Actually, it is true in general $\Delta_K$ divides $\Delta_L$ in a tower $L/K/\QQ$.) \subsection{Step 2: Reformulation} Now we are going to prove: \begin{theorem} [Quadratic reciprocity, equivalent formulation] For distinct odd primes $p$, $q$ we have \[ \left( \frac pq \right) = \left( \frac{q^\ast}{p} \right). \] \end{theorem} \begin{exercise} Using the fact that $\left( \frac{-1}{p} \right) = (-1)^{\frac{p-1}{2}}$, show that this is equivalent to quadratic reciprocity as we know it. \end{exercise} We look at the rational prime $p$ in $\ZZ$. Either it splits into two in $K$ or is inert; either way let $\kp$ be a prime factor in the resulting decomposition (so $\kp$ is either $p \cdot \OO_K$ in the inert case, or one of the primes in the split case). Then let ${\kP}$ be above $\kp$. It could possibly also split in $K$: the picture looks like \begin{center} \begin{tikzcd} \OO_L = \ZZ[\zeta_q] & \supset & {\kP} \ar[r, dotted] & \ZZ[\zeta_p]/{\kP} \cong \FF_{p^f} \\ \OO_K = \ZZ[\frac{1+\sqrt{q^\ast}}{2}] & \supset & \kp \ar[r, dotted] & \FF_p \text{ or } \FF_{p^2} \\ \ZZ & \supset & (p) \ar[r, dotted] & \FF_p \end{tikzcd} \end{center} \begin{ques} Why is $p$ not ramified in either $K$ or $L$? \end{ques} \subsection{Step 3: Introducing the Frobenius} Now, we take the Frobenius \[ \sigma_p = \Frob_{\kP} \in \Gal(L/\QQ). \] We claim that \[ \Frob_{\kP} \in H \iff \text{$p$ splits in $K$}. \] To see this, note that $\Frob_{\kP}$ is in $H$ if and only if it acts as the identity on $K$. But $\Frob_{\kP} \restrict{K}$ is $\Frob_\kp$! So \[ \Frob_{\kP} \in H \iff \Frob_\kp = \id_K. \] Finally note that $\Frob_\kp$ has order $1$ if $p$ splits ($\kp$ has inertial degree $1$) and order $2$ if $p$ is inert. This completes the proof of the claim. \subsection{Finishing up} We already know by \Cref{lem:cyclo_frob} that $\Frob_{\kP} = \sigma_p \in H$ if and only if $p$ is a quadratic residue. On the other hand, \begin{exercise} Show that $p$ splits in $\OO_K = \ZZ[\frac12(1+\sqrt{q^\ast})]$ if and only if $\left( \frac{q^\ast}{p} \right) = 1$. (Use the factoring algorithm. You need the fact that $p \neq 2$ here.) \end{exercise} In other words \[ \left( \frac pq \right) = 1 \iff \sigma_p \in H \iff \text{$p$ splits in $\ZZ\left[ \tfrac12(1+\sqrt{q^\ast}) \right]$} \iff \left( \frac{q^\ast}{p} \right) = 1. \] This completes the proof. \section{Frobenius elements control factorization} \prototype{$\Frob_\kp$ controlled the splitting of $p$ in the proof of quadratic reciprocity; the same holds in general.} In the proof of quadratic reciprocity, we used the fact that Frobenius elements behaved well with restriction in order to relate the splitting of $p$ with properties of $\Frob_\kp$. In fact, there is a much stronger statement for any intermediate field $\QQ \subseteq E \subseteq K$ which works even if $E/\QQ$ is not Galois. It relies on the notion of a \emph{factorization pattern}. Here is how it goes. Set $n = [E:\QQ]$, and let $p$ be a rational prime unramified in $K$. Then $p$ can be broken in $E$ as \[ p \cdot \OO_E = \kp_1 \kp_2 \dots \kp_g \] with inertial degrees $f_1$, \dots, $f_g$: (these inertial degrees might be different since $E/\QQ$ isn't Galois). The numbers $f_1 + \dots + f_g = n$ form a partition of the number $n$. For example, in the quadratic reciprocity proof we had $n = 2$, with possible partitions $1 + 1$ (if $p$ split) and $2$ (if $p$ was inert). We call this the \vocab{factorization pattern} of $p$ in $E$. Next, we introduce a Frobenius $\Frob_{\kP}$ above $(p)$, all the way in $K$; this is an element of $G = \Gal(K/\QQ)$. Then let $H$ be the group corresponding to the field $E$. Diagram: \begin{center} \begin{tikzcd} K \ar[r, leftrightarrow] \ar[d, dash] & \{1\} \ar[d, dash] & \Frob_{\kP} \\ E \ar[d, dash, "n"'] \ar[r, leftrightarrow] & H \ar[d, dash, "n"] & \kp_1 \dots \kp_g \ar[d, dash] & f_1 + \dots + f_g = n \\ \QQ \ar[r, leftrightarrow] & G & (p) \end{tikzcd} \end{center} Then $\Frob_{\kP}$ induces a \emph{permutation} of the $n$ left cosets $gH$ by left multiplication (after all, $\Frob_{\kP}$ is an element of $G$ too!). Just as with any permutation, we may look at the resulting cycle decomposition, which has a natural ``cycle structure'': a partition of $n$. \begin{center} \begin{asy} size(8cm); pen tg = heavyred; // "times g" pen pointpen = lightblue; pair A = Drawing("g_1H", dir(80), dir(80), pointpen); pair B = Drawing("g_2H", A*dir(120), A*dir(120), pointpen); pair C = Drawing("g_3H", A*dir(240), A*dir(240), pointpen); draw(A--B, dashed + pointpen, EndArrow, Margin(2,2)); draw(B--C, dashed + pointpen, EndArrow, Margin(2,2)); draw(C--A, dashed + pointpen, EndArrow, Margin(2,2)); label("$\times g$", midpoint(A--B), A+B, tg); label("$\times g$", midpoint(B--C), B+C, tg); label("$\times g$", midpoint(C--A), C+A, tg); label("$3$", origin, origin, pointpen); add(shift( (-3.2,0.1) ) * CC()); label("$g = \operatorname{Frob}_{\mathfrak P}$", (-1.7,1.7), origin, tg); pointpen = heavygreen; pair W = Drawing("g_4H", dir(50), dir(50), pointpen); pair X = Drawing("g_5H", W*dir(90), W*dir(90), pointpen); pair Y = Drawing("g_6H", W*dir(180), W*dir(180), pointpen); pair Z = Drawing("g_7H", W*dir(270), W*dir(270), pointpen); draw(W--X, dashed + pointpen, EndArrow, Margin(2,2)); draw(X--Y, dashed + pointpen, EndArrow, Margin(2,2)); draw(Y--Z, dashed + pointpen, EndArrow, Margin(2,2)); draw(Z--W, dashed + pointpen, EndArrow, Margin(2,2)); defaultpen(red); label("$\times g$", W--X, W+X, tg); label("$\times g$", X--Y, X+Y, tg); label("$\times g$", Y--Z, Y+Z, tg); label("$\times g$", Z--W, Z+W, tg); label("$4$", origin, origin, pointpen); label("$\boxed{n = 7 = 3+4}$", (-2,-1.8), origin, black); \end{asy} \end{center} The theorem is that these coincide: \begin{theorem} [Frobenius elements control decomposition] \label{thm:frob_control_decomp} Let $\QQ \subseteq E \subseteq K$ an extension of number fields and assume $K/\QQ$ is Galois (though $E/\QQ$ need not be). Pick an unramified rational prime $p$; let $G = \Gal(K/\QQ)$ and $H$ the corresponding intermediate subgroup. Finally, let $\kP$ be a prime above $p$ in $K$. Then the \emph{factorization pattern} of $p$ in $E$ is given by the \emph{cycle structure} of $\Frob_{\kP}$ acting on the left cosets of $H$. \end{theorem} Often, we take $E = K$, in which case this is just asserting that the decomposition of the prime $p$ is controlled by a Frobenius element over it. An important special case is when $E = \QQ(\alpha)$, because as we will see it is let us determine how the minimal polynomial of $\alpha$ factors modulo $p$. To motivate this, let's go back a few chapters and think about the Factoring Algorithm. Let $\alpha$ be an algebraic integer and $f$ its minimal polynomial (of degree $n$). Set $E = \QQ(\alpha)$ (which has degree $n$ over $\QQ$). Suppose we're lucky enough that $\OO_E = \ZZ[\alpha]$, i.e.\ that $E$ is monogenic. Then we know by the Factoring Algorithm, to factor any $p$ in $E$, all we have to do is factor $f$ modulo $p$, since if $f = f_1^{e_1} \dots f_g^{e_g} \pmod p$ then we have \[ (p) = \prod_i \kp_i = \prod_i (f_i(\alpha), p)^{e_i}. \] This gives us complete information about the ramification indices and inertial degrees; the $e_i$ are the ramification indices, and $\deg f_i$ are the inertial degrees (since $\OO_E / \kp_i \cong \FF_p[X] / (f_i(X))$). In particular, if $p$ is unramified then all the $e_i$ are equal to $1$, and we get \[ n = \deg f = \deg f_1 + \deg f_2 + \dots + \deg f_g. \] Once again we have a partition of $n$; we call this the \vocab{factorization pattern} of $f$ modulo $p$. So, to see the factorization pattern of an unramified $p$ in $\OO_E$, we just have to know the factorization pattern of the $f \pmod p$. Turning this on its head, if we want to know the factorization pattern of $f \pmod p$, we just need to know how $p$ decomposes. And it turns out these coincide even without the assumption that $E$ is monogenic. \begin{theorem}[Frobenius controls polynomial factorization] \label{thm:factor_poly_frob} Let $\alpha$ be an algebraic integer with minimal polynomial $f$, and let $E = \QQ(\alpha)$. Then for any prime $p$ unramified in the splitting field $K$ of $f$, the following coincide: \begin{enumerate}[(i)] \ii The factorization pattern of $p$ in $E$. \ii The factorization pattern of $f \pmod p$. \ii The cycle structure associated to the action of $\Frob_{\kP} \in \Gal(K/\QQ)$ on the roots of $f$, where $\kP$ is above $p$ in $K$. \end{enumerate} \end{theorem} \begin{example}[Factoring $x^3-2 \pmod 5$] Let $\alpha = \cbrt2$ and $f = x^3-2$, so $E = \QQ(\cbrt2)$. Set $p=5$ and finally, let $K = \QQ(\cbrt2, \omega)$ be the splitting field. Setup: \begin{center} \begin{tikzcd} K = \QQ(\cbrt2, \omega) \ar[d, dash, "2"'] & \kP \ar[d, dash] & x^3-2 = (x-\cbrt2)(x-\cbrt2\omega)(x-\cbrt2\omega^2) \\ E = \QQ(\sqrt[3]{2}) \ar[d, dash, "3"'] & \kp \ar[d, dash] & x^3-2 = (x-\cbrt2)(x^2+\cbrt2x+\cbrt4) \\ \QQ & (5) & x^3-2 \text{ irreducible over } \QQ \end{tikzcd} \end{center} The three claimed objects now all have shape $2+1$: \begin{enumerate}[(i)] \ii By the Factoring Algorithm, we have $(5) = (5, \cbrt2-3)(5, 9+3\cbrt2+\cbrt4)$. \ii We have $x^3-2 \equiv (x-3)(x^2+3x+9) \pmod 5$. \ii We saw before that $\Frob_{\kP} = (\bullet)(\bullet \; \bullet)$. \end{enumerate} \end{example} \begin{proof}[Sketch of Proof] Letting $n = \deg f$. Let $H$ be the subgroup of $G = \Gal(K/\QQ)$ corresponding to $E$, so $[G:E] = n$. Pictorially, we have \begin{center} \begin{tikzcd} K \ar[d, dash] & \{1\} \ar[d, dash] & \kP \ar[d, dash] \\ E = \QQ(\alpha) \ar[d, dash] & H \ar[d, dash] & \kp \ar[d, dash] \\ \QQ & G & (p) \end{tikzcd} \end{center} We claim that (i), (ii), (iii) are all equivalent to \begin{center} (iv) The pattern of the action of $\Frob_{\kP}$ on the $G/H$. \end{center} In other words we claim the cosets correspond to the $n$ roots of $f$ in $K$. Indeed $H$ is just the set of $\tau \in G$ such that $\tau(\alpha)=\alpha$, so there's a bijection between the roots and the cosets $G/H$ by $\tau H \mapsto \tau(\alpha)$. Think of it this way: if $G = S_n$, and $H = \{\tau : \tau(1) = 1\}$, then $G/H$ has order $n! / (n-1)! = n$ and corresponds to the elements $\{1, \dots, n\}$. So there is a natural bijection from (iii) to (iv). The fact that (i) is in bijection to (iv) was the previous theorem, \Cref{thm:frob_control_decomp}. The correspondence (i) $\iff$ (ii) is a fact of Galois theory, so we omit the proof here. \end{proof} All this can be done in general with $\QQ$ replaced by $F$; for example, in \cite{ref:lenstra_chebotarev}. \section{Example application: IMO 2003 problem 6} As an example of the power we now have at our disposal, let's prove: \begin{center} \begin{minipage}{4.5cm} \includegraphics[width=4cm]{media/IMO-2003-logo.png} \end{minipage}% \begin{minipage}{10cm} \textbf{Problem 6}. Let $p$ be a prime number. Prove that there exists a prime number $q$ such that for every integer $n$, the number $n^p-p$ is not divisible by $q$. \end{minipage} \end{center} We will show, much more strongly, that there exist infinitely many primes $q$ such that $X^p-p$ is irreducible modulo $q$. \begin{proof}[Solution] Okay! First, we draw the tower of fields \[ \QQ \subseteq \QQ(\sqrt[p]{p}) \subseteq K \] where $K$ is the splitting field of $f(x) = x^p-p$. Let $E = \QQ(\sqrt[p]{p})$ for brevity and note it has degree $[E:\QQ] = p$. Let $G = \Gal(K/\QQ)$. \begin{ques} Show that $p$ divides the order of $G$. (Look at $E$.) \end{ques} Hence by Cauchy's theorem (\Cref{thm:cauchy_group}, which is a purely group-theoretic fact) we can find a $\sigma \in G$ of order $p$. By Chebotarev, there exist infinitely many rational (unramified) primes $q \neq p$ and primes $\kQ \subseteq \OO_K$ above $q$ such that $\Frob_\kQ = \sigma$. (Yes, that's an uppercase Gothic $Q$. Sorry.) We claim that all these $q$ work. By \Cref{thm:factor_poly_frob}, the factorization of $f \pmod q$ is controlled by the action of $\sigma = \Frob_\kQ$ on the roots of $f$. But $\sigma$ has prime order $p$ in $G$! So all the lengths in the cycle structure have to divide $p$. Thus the possible factorization patterns of $f$ are \[ p = \underbrace{1 + 1 + \dots + 1}_{\text{$p$ times}} \quad\text{or}\quad p = p. \] So we just need to rule out the $p = 1 + \dots + 1$ case now: this only happens if $f$ breaks into linear factors mod $q$. Intuitively this edge case seems highly unlikely (are we really so unlucky that $f$ factors into \emph{linear} factors when we want it to be irreducible?). And indeed this is easy to see: this means that $\sigma$ fixes all of the roots of $f$ in $K$, but that means $\sigma$ fixes $K$ altogether, and hence is the identity of $G$, contradiction. \end{proof} \begin{remark} In fact $K = \QQ(\sqrt[p]{p}, \zeta_p)$, and $\left\lvert G \right\rvert = p(p-1)$. With a little more group theory, we can show that in fact the density of primes $q$ that work is $\frac 1p$. \end{remark} \section\problemhead \begin{problem} Show that for an odd prime $p$, \[ \left( \frac 2p \right) = (-1)^{\frac 18(p^2-1)}. \] \begin{hint} Modify the end of the proof of quadratic reciprocity. \end{hint} \begin{sol} It is still true that \[ \left( \frac 2q \right) = 1 \iff \sigma_2 \in H \iff \text{$2$ splits in $\ZZ\left[ \tfrac12(1+\sqrt{q^\ast}) \right]$}. \] Now, $2$ splits in the ring if and only if $t^2 - t - \tfrac14(1-q^\ast)$ factors mod $2$. This happens if and only if $q^\ast \equiv 1 \pmod 8$. One can check this is exactly if $q \equiv \pm 1 \pmod 8$, which gives the conclusion. \end{sol} \end{problem} \begin{problem} Let $f$ be a nonconstant polynomial with integer coefficients. Suppose $f \pmod p$ splits completely into linear factors for all sufficiently large primes $p$. Show that $f$ splits completely into linear factors. \end{problem} \begin{dproblem} [Dirichlet's theorem on arithmetic progressions] Let $a$ and $m$ be relatively prime positive integers. Show that the density of primes $p \equiv a \pmod m$ is exactly $\frac{1}{\phi(m)}$. \begin{hint} Chebotarev Density on $\QQ(\zeta_m)$. \end{hint} \begin{sol} Let $K = \Gal(\QQ(\zeta_m)/\QQ)$. One can show that $\Gal(K/\QQ) \cong \Zm m$ exactly as before. In particular, $\Gal(K/\QQ)$ is abelian and therefore its conjugacy classes are singleton sets; there are $\phi(m)$ of them. As long as $p$ is sufficiently large, it is unramified and $\sigma_p = \Frob_\kp$ for any $\kp$ above $p$ (as $m$th roots of unity will be distinct modulo $p$; differentiate $x^m-1$ mod $p$ again). \end{sol} \end{dproblem} \begin{problem} Let $n$ be an odd integer which is not a prime power. Show that the $n$th cyclotomic polynomial is not irreducible modulo \emph{any} rational prime. % http://mathoverflow.net/questions/12366/how-many-primes-stay-inert-in-a-finite-non-cyclic-extension-of-number-fields \end{problem} \begin{problem} [Putnam 2012 B6] \yod Let $p$ be an odd prime such that $p \equiv 2 \pmod 3$. Let $\pi$ be a permutation of $\FF_p$ by $\pi(x) = x^3 \pmod p$. Show that $\pi$ is even if and only if $p \equiv 3 \pmod 4$. \begin{hint} By primitive roots, it's the same as the action of $\times 3$ on $\Zcc{p-1}$. Let $\zeta$ be a $(p-1)$st root of unity. Take $d = \prod_{i < j} (\zeta^i - \zeta^j)$, think about $\QQ(d)$, and figure out how to act on it by $x \mapsto x^3$. \end{hint} \begin{sol} This solution is by David Corwin. By primitive roots, it's the same as the action of $\times 3$ on $\Zcc{p-1}$. Let $\zeta$ be a $(p-1)$st root of unity. Consider \[ d = \prod_{0 \le i < j < p-1} (\zeta^i - \zeta^j). \] This is the square root of the discriminant of the polynomial $X^{p-1}-1$; in other words $d^2 \in \ZZ$. In fact, by elementary methods one can compute \[ (-1)^{\binom{p-1}{2}} d^2 = -(p-1)^{p-1} \] Now take the extension $K = \QQ(d)$, noting that \begin{itemize} \ii If $p \equiv 3 \pmod 4$, then $d = (p-1)^{\half(p-1)}$, so $K = \QQ$. \ii If $p \equiv 1 \pmod 4$, then $d = i(p-1)^{\half(p-1)}$, so $K = \QQ(i)$. \end{itemize} Either way, in $\OO_K$, let $\kp$ be a prime ideal above $(3) \subseteq \OO_K$. Let $\sigma = \Frob_\kp$ then be the unique element such that $\sigma(x) = x^3 \pmod{\kp}$ for all $x$. Then, we observe that \[ \sigma(d) \equiv \prod_{0 \le i < j < p-1} (\zeta^{3i} - \zeta^{3j}) \equiv \begin{cases} +d & \text{if $\pi$ is even} \\ -d & \text{if $\pi$ is odd} \end{cases} \pmod{\kp}. \] Now if $K = \QQ$, then $\sigma$ is the identity, thus $\sigma$ even. Conversely, if $K = \QQ(i)$, then $3$ does not split, so $\sigma(d) = -d$ (actually $\sigma$ is complex conjugation) thus $\pi$ is odd. Note the condition that $p \equiv 2 \pmod 3$ is used only to guarantee that $\pi$ is actually a permutation (and thus $d \neq 0$); it does not play any substantial role in the solution. \end{sol} \end{problem}